Docsity
Docsity

Prepare for your exams
Prepare for your exams

Study with the several resources on Docsity


Earn points to download
Earn points to download

Earn points by helping other students or get them with a premium plan


Guidelines and tips
Guidelines and tips

Pregnancy and Childbirth: Risks and Interventions, Exams of Nursing

Answers to multiple-choice questions on various aspects of pregnancy, including fetal development, prenatal care, and labor. Topics covered include obesity, heart defects, respiratory distress, cat feces exposure, support for pregnant women, folic acid supplementation, weight gain, blood pressure, soft cheese and listeria, gestational hypertension, smoking, back pain, emotional changes, thrombocytopenia, preterm labor, vaginal infections, pelvic pressure, impaired attachment, circumcision, macrosomic babies, and urinary tract infection in older women.

Typology: Exams

2023/2024

Available from 06/04/2024

coursehero
coursehero 🇺🇸

4.3

(40)

813 documents

1 / 206

Toggle sidebar

Related documents


Partial preview of the text

Download Pregnancy and Childbirth: Risks and Interventions and more Exams Nursing in PDF only on Docsity! 1 OB NURS 306 OB-Study Guide by chapters Ch1-to-Ch19 goodstuf Chapter 1: Trends and Issues Multiple Choice 1. Since 1995 there has been a significant decrease in the rate of infant death related to which of the following: a. Disorders associated with short gestation and low birth weight b. Accidents c. Sudden infant death d. Newborns affected by complications of placenta, cord, and membranes ANS: c 2. Tobacco use during pregnancy is associated with adverse effects on the unborn infant such as intrauterine growth restriction, preterm births, and respiratory problems. By race, which has the highest percentages of smokers? a. American Indian and Alaskan Natives b. Asian or Pacific Islanders c. Non-Hispanic blacks d. Non-Hispanic whites ANS: a 3. Which of the following women is at the highest risk for health disparity? Feedback a. The rates of prematurity and low birth weight are increasing. b. The rates of accidents have increased. c. Correct. The rate of infant death related to SIDS has decreased from 87.1 to 47.2. The decrease in rate is partially attributed to placing infants on their backs when sleeping. d. The rates of newborns affected by complications of placenta, cord, and membranes have increased. Feedback a. 36% of American Indian and Native American women are cigarette smokers. b. 4.3% of Asian or Pacific Islander women are cigarette smokers. c. 17.1% of non-Hispanic black women are cigarette smokers. d. 19.6% of non-Hispanic white women are cigarette smokers. 2 a. A white, middle-class, 16-year-old woman 5 ANS: d Feedback a. Adaptation rather than cure is the goal of nursing. b. Nursing does not define the value of a person by his or her utility. c. The Code of Ethics outlines the nursing profession’s nonnegotiable ethical standard. d. Respect for the inherent dignity, worth, and uniqueness of every individual is part of the Code of Ethics. 3. Evidence-based practice is the integration of the best: a. Randomized clinical trials, clinical expertise, and patients’ requestsb. Research evidence, clinical expertise, and patients’ valuesc. Quantitative research, clinical expertise, and patients’ preferencesd. Research findings, clinical experience, and patients’ preferences ANS: b Feedback a. Evidence-based practice is the use of evidence that may include research beyond randomized clinical trials. b. These elements are the accepted definition of evidence-based practice. c. Qualitative research, as well as quantitative research, contributes to evidence-based practice. d. Clinical expertise, as well as clinical experience, defines evidence-based practice. Multiple Response 4. Infants whose mothers were obese during pregnancy are at higher risk for which of the following? (Select all that apply.) a. Childhood diabetes b. Heart defects c. Hypospadias d. Respiratory distress ANS: a, b, c Fetuses and/or infants of women who were obese during pregnancy are at higher risk for spina bifida, health defects, anorectal atresia, hypospadias, intrauterine fetal death, birth injuries related to macrosomia, and childhood obesity and diabetes. 6 Chapter 3: Genetics, Conception, Fetal Development, and Reproductive Technology Multiple Choice 1. The color of a person’s hair is an example of which of the following? a. Genome b. Sex-link inheritance. C. Genotype d. Phenotype ANS: d Feedback a. Genome is an organism’s complete set of DNA. b. Sex-link inheritance refers to genes or traits that are located only on the X chromosome. c. Genotype refers to a person’s genetic makeup. d. Correct. Phenotype refers to how genes are outwardly expressed, such as eye color, hair color, and height. 2. Which of the following statements by a pregnant woman indicates she needs additional teaching on ways to reduce risks to her unborn child from the potential effects of exposure to toxoplasmosis? a. “I will avoid rare lamb.” b. “I will wear a mask when cleaning my cat’s litter box.” c. “I understand that exposure to toxoplasmosis can cause blindness in the baby.” d. “I will avoid rare beef.” ANS: b Feedback a. Exposure occurs when the protozoan parasite found in cat feces and uncooked or rare beef and lamb is ingested. b. Correct. Pregnant women and women who are attempting pregnancy should avoid contact with cat feces. Exposure occurs when the protozoan parasite found in cat feces and uncooked or rare beef and lamb is ingested. Wearing a mask will not decrease the risk through ingestion of the parasite. c. Exposure to toxoplasmosis can cause fetal death, mental retardation, and blindness. d. Exposure occurs when the protozoan parasite found in cat feces and uncooked or rare beef and lamb is ingested. 3. The fetal circulatory structure that connects the pulmonary artery with the descending aorta is known as which of the following? a. Ductus venosus b. Foramen ovale c. Ductus arteriosus d. Internal iliac artery 7 ANS: c Feedback a. The ductus venosus connects the umbilical vein to the inferior vena cava. b. The foramen ovale is the opening between the right and left atria. c. Correct. d. The internal iliac artery connects the external iliac artery to the umbilical artery. 4. A woman at 40 weeks’ gestation has a diagnosis of oligohydramnios. Which of the following statements related to oligohydramnios is correct? a. It indicates that there is a 25% increase in amniotic fluid. b. It indicates that there is a 25% reduction of amniotic fluid. c. It indicates that there is a 50% increase in amniotic fluid. d. It indicates that there is a 50% reduction of amniotic fluid. ANS: d Feedback a. Oligohydramnios is a decrease, not an increase in amniotic fluid. b. Oligohydramnios is a 50% reduction in amniotic fluid. c. Oligohydramnios is a decrease, not an increase in amniotic fluid. d. Correct. Oligohydramnios refers to a decreased amount of amniotic fluid of less than 500 mL at term or 50% reduction of normal amounts. 5. A diagnostic test commonly used to assess problems of the fallopian tubes is: a. Endometrial biopsy b. Ovarian reserve testing c. Hysterosalpingogram d. Screening for sexually transmitted infections ANS: c Feedback a. Endometrial biopsy provides information on the response of the uterus to hormonal signals. b. Ovarian reserve testing is used to assess ovulatory functioning. c. Correct. Hysterosalpingogram provides information on the endocervical canal, uterine cavity, and fallopian tubes. d. STIs can cause adhesions within the fallopian tubes, but screening cannot confirm that adhesions are present. KEY: Integrated Process: Teaching and Learning | Cognitive Level: Knowledge | Content Area: Maternity | Client Need: Safe and Effective Care Environment | Difficulty Level: Moderate 10 b. Blood returns to the placenta via the umbilical arteries. c. Most of the blood bypasses the pulmonary system. The blood that does enter the pulmonary system returns to the left atrium. d. There is no duct between the umbilical arteries. 11. The clinic nurse knows that the part of the endometrial cycle occurring from ovulation to just prior to menses is known as the: a. Menstrual phase b. Proliferative phase c. Secretory phase d. Ischemic phase ANS: c 12. A clinic nurse explains to the pregnant woman that the amount of amniotic fluid present at 24 weeks’ gestation is approximately: a. 500 mL b. 750 mL c. 800 mL d. 1000 mL ANS: c Amniotic fluid first appears at about 3 weeks. There are approximately 30 mL of amniotic fluid present at 10 weeks’ gestation, and this amount increases to approximately 800 mL at 24 weeks’ gestation. After that time, the total fluid volume remains fairly stable until it begins to decrease slightly as the pregnancy reaches term. 13. Information provided by the nurse that addresses the function of the amniotic fluid is that the amniotic fluid helps the fetus to maintain a normal body temperature and also: a. Facilitates asymmetrical growth of the fetal limbs b. Cushions the fetus from mechanical injury Feedback a. The menstrual phase is the time of vaginal bleeding, approximately days 1 to 6. b. The proliferative phase ends the menses through ovulation, approximately days 7 to 14. c. The secretory phases occurs from the time of ovulation to the period just prior to menses, or approximately days 15 to 26. d. The ischemic phase occurs from the end of the secretory phase to the onset of menstruation, approximately days 27 to 28. 11 c. Promotes development of muscle tone d. Promotes adherence of fetal lung tissue ANS: b 14. During preconception counseling, the clinic nurse explains that the time period when the fetus is most vulnerable to the effects of teratogens occurs from: a. 2 to 8 weeks b. 4 to12 weeks c. 5 to 10 weeks d. 6 to 15 weeks ANS: a The period of organogenesis lasts from approximately the second until the eighth week of gestation during which time the embryo undergoes rapid growth and differentiation. During organogenesis, the embryo is extremely vulnerable to teratogens such as medications, alcohol, tobacco, caffeine, illegal drugs, radiation, heavy metals, and maternal (TORCH) infections. Structural fetal defects are most likely to occur during this period because exposure to teratogens either before or during a critical period of development of an organ can cause a malformation. 15. A major fetal development characteristic at 16 weeks’ gestation is: a. The average fetal weight is 450 grams b. Lanugo covers entire body c. Brown fat begins to develop d. Teeth begin to form ANS: d 16. Karen, a 26-year-old woman, has come for preconception counseling and asks about caring for her cat as she has heard that she “should not touch the cat during pregnancy.” The clinic nurse’s best response is: a. It is best if someone other than you change the cat’s litter pan during pregnancy so that you Feedback a. Amniotic fluid allows for symmetrical fetal growth. b. Amniotic fluid cushions the fetus from mechanical injury. c. Amniotic fluid does not promote muscle tone. d. Amniotic fluid prevents adherence of the amnion to the fetus. Feedback a. The average fetal weight at 16 weeks is 200 grams. b. Lanugo is present on the head. c. Brown fat begins to develop at 20 weeks. d. This is the correct answer. 12 have no risk of toxoplasmosis during pregnancy. b. It is important to have someone else change the litter pan during pregnancy and also avoid consuming raw vegetables. c. Have you had any “flu-like” symptoms since you got your cat? If so, you may have already had toxoplasmosis and there is nothing to worry about. d. Toxoplasmosis is a concern during pregnancy, so it is important to have someone else change the cat’s litter pan and also to avoid consuming uncooked meat. ANS: d Feedback a. The nurse should also explain that the patient should not eat uncooked meat as it is a potential source for toxoplasmosis. b. Raw vegetables are not a source for toxoplasmosis. c. This is not an accurate way to diagnose if the woman has had toxoplasmosis. d. Women need to be aware that Toxoplasma gondii, a single-celled parasite, is responsible for the infection toxoplasmosis. The majority of individuals who become infected with toxoplasmosis are asymptomatic, although when present, symptoms are described as “flu like” and include glandular pain and enlargement and myalgia. Severe toxoplasmosis infection may cause damage to the fetal brain, eyes, or other organs. Toxoplasmosis is usually acquired by consuming raw or poorly cooked meat that has been contaminated with T. gondii. Toxoplasmosis may also be acquired through close contact with feces from an infected animal (usually cats) or soil that has been contaminated with T. gondii. 17. A couple who has sought infertility counseling has been told that the man’s sperm count is very low. The nurse advises the couple that spermatogenesis is impaired when which of the following occur? a. The testes are overheated. b. The vas deferens is ligated. c. The prostate gland is enlarged. d. The flagella are segmented. ANS: a Feedback a. Spermatogenesis occurs in the testes. High 15 and development of the embryo/fetus. ANS: teratogen Teratogens (drugs, radiation, and infectious agents that can cause development of abnormal structures in an embryo) and a variety of internal and external developmental events may cause structural and functional defects. 25. is when sperm and oocytes are mixed outside the woman’s body and then placed into the fallopian tube via laparoscopy. ANS: Gamete intrafallopian transfer Gamete intrafallopian transfer, also referred to as GIFT, is used when there is a history of failed infertility treatment for anovulation, or unexplained infertility, or low sperm count. Multiple Response 26. A woman seeks care at an infertility clinic. Which of the following tests may this woman undergo to determine what, if any, infertility problem she may have? (Select all that apply.) a. Chorionic villus sampling b. Endometrial biopsy c. Hysterosalpingogram d. Serum FSH analysis ANS. b, c, d 27. A couple who has been attempting to become pregnant for 5 years is seeking assistance from an infertility clinic. The nurse assesses the clients’ emotional responses to their infertility. Which of the following responses would the nurse expect to find? (Select all that apply.) a. Anger at others who have babies. b. Feelings of failure because they cannot make a baby. c. Sexual excitement because they want to conceive a baby. d. Guilt on the part of one partner because he or she is unable to give the other a baby. ANS: a, b, d Feedback a. Infertile couples often feel anger toward Feedback a. Chorionic villus sampling is done to assess for genetic disorders of the fetus. b. Endometrial biopsy is performed about 1 week following ovulation to detect the endometrium’s response to progesterone. c. Hysterosalpingogram is used to determine if fallopian tubes are patent. d. Serum FSH levels are used to assess ovarian function. 16 couples who have babies. b. Infertile couples often express feelings of personal failure. c. Infertile couples undergoing infertility testing and treatment often express an aversion to sex. d. Guilt is often expressed by the couple. 28. Which of the following places a couple at higher risk for conceiving a child with a genetic abnormality? (Select all that apply.) a. Maternal age over 35 years b. Partner who has a genetic disorder c. Maternal type 1 diabetes d. Paternal heart disease ANS: a, b Fertility decreases after 35 years. A partner contributes half of the chromosomal makeup, and genetic disorders can be inherited. Maternal diabetes can have an effect on the fetus/neonate, such as causing complications such as macrosomia and hypoglycemia, but these are not genetic disorders. Paternal heart disease can place the neonate at risk for heart disease later in life, but this is not referred to as a genetic disorder, such as is trisomy 21 and hemophilia. 29. The ovarian cycle includes which of the following phases? (Select all that apply.) a. Follicular phase b. Secretory phase c. Ovulatory phase d. Luteal phase e. Menstrual phase ANS: a, c, d Follicular phase, ovulatory phase, and luteal phase are part of the ovarian cycle. Secretory and menstrual phases are part of the endometrial cycle. 30. A couple is undergoing an infertility workup. The semen analysis indicates a decreased number of sperm and immature sperm. Which of the following factors can have a potential effect on sperm maturity? (Select all that apply.) a. The man rides a bike to and from work each day. b. The man takes a calcium channel blocker for the treatment of hypertension. c. The man drinks 6 cups of coffee a day. d. The man was treated for prostatitis 12 months ago and has been symptom free since treatment. ANS: a, b The daily riding of a bike can be the cause of prolonged heat exposure to the testicles. Prolonged heat exposure is a gonadotoxin. A number of medications, such as calcium channel blockers, can have an effect on sperm production. Coffee has not been associated with low sperm counts. Prostatitis or other infections within the last 3 months may have an effect on the sperm analysis. This man’s episode of prostatitis was 12 months prior. 17 KEY: Integrated Process: Teaching and Learning | Cognitive Level: Application | Content Area: Maternity | Client Need: Health Promotion and Maintenance | Difficulty Level: Moderate 31. The clinic nurse recognizes that pregnant women who are in particular need of support are those who (select all that apply): a. Are experiencing a second pregnancy b. Are awaiting genetic testing results c. Are experiencing a first pregnancy d. Are trying to conceal this pregnancy as long as possible ANS: b, d A second pregnancy is not an indication of a woman in need of additional support. A support system may be lacking for women who are trying to conceal a pregnancy or for women who are trying to keep the news of their pregnancy from relatives or friends until results from genetic tests are known. These individuals may need additional support from their nurses and other health-care providers, as they are placed in a powerless situation while awaiting results and face a pregnancy that may be in jeopardy. Chapter 4: Physiological Aspects of Antepartum Care Multiple Choice 1. Folic acid supplementation during pregnancy is to:a. Improve the bone density of pregnant women b. Decrease the incidence of neural tube defects in the fetus c. Decrease the incidence of Down syndrome in the fetusd. Improve calcium uptake in pregnant women ANS: b Feedback a. Folic acid is not related to bone density. b. Correct. The use of folic acid has decreased the incidence of neural tube defects by 50%. c. The use of folic acid is not associated with a reduction in Down syndrome. d. Folic acid is not related to calcium uptake in women. 2. The positive signs of pregnancy are: a. All physiological and anatomical changes of pregnancy b. All subjective signs of pregnancy c. All those physiological changes perceived by the woman herself d. The objective signs of pregnancy that can only be attributed to the fetus ANS: d Feedback a. Physiological and anatomical changes of 20 third trimester. c. Pregnant women need to increase their fluid intake during pregnancy, and dysuria and urgency are abnormal. d. Assessment of fetal position and station is not an appropriate response to reported signs and symptoms of a urinary tract infection. 7. At the end of her 32-week prenatal visit, a woman reports discomfort with intercourse and tells you shyly that she wants to maintain a sexual relationship with her partner. The best response is to: a. Reassure woman/couple of normalcy of response b. Suggest alternative positions for sexual intercourse and alternative sexual activity to sexual intercourse c. Recommend cessation of intercourse until after delivery due to advanced gestation d. Suggest woman discuss this with her care provider at her next appointment ANS: b Feedback a. Although this is a normal response, providing reassurance is not enough. Further intervention is indicated. b. Although shy to discuss this, she wants to maintain a sexual relationship with her partner. Suggesting alternative positions for sexual intercourse and alternative sexual activity to sexual intercourse provides the woman with information to maintain sexual relations. c. She wants to maintain a sexual relationship with her partner, and there are no contraindications to intercourse during a healthy pregnancy. d. The patient is seeking out information and to defer her to her care provider at her next appointment is inappropriate. Additionally, she may not be comfortable discussing this with anyone else. 8. The clinic nurse talks to a 30-year-old woman at 34 weeks’ gestation who complains of having difficulty sleeping. Jayne has noticed that getting back to sleep after she has been up at night is difficult. The nurse’s best response is: a. “This is abnormal; it is important that you describe this problem to the doctor.” b. “This is normal, and many women have this same problem during pregnancy; try napping for several hours each morning and afternoon.” c. “This is abnormal; tell the doctor about this problem because diagnostic testing may be necessary.” d. “This is normal in pregnancy, particularly during the third trimester when you also feel fetal movement at night; try napping once a day.” 21 ANS: d Feedback a. This sleep pattern is a normal finding. b. Sleeping for several hours in the morning and afternoon would contribute to further sleep disturbances at night. c. This sleep pattern is a normal finding. d. Pregnancy sleep patterns are characterized by reduced sleep efficiency, fewer hours of night sleep, frequent awakenings, and difficulty going to sleep. Nurses can advise patients that afternoon napping may help alleviate the fatigue associated with the sleep alterations. 9. A 26-year-old woman at 29 weeks’ gestation experienced epigastric pain following the consumption of a large meal of fried fish and onion rings. The pain resolved a few hours later. The most likely diagnosis for this symptom is: a. Cholelithiasis b. Influenza c. Urinary tract infection d. Indigestion ANS: a Feedback a. The progesterone-induced prolonged emptying time of bile from the gallbladder, combined with elevated blood cholesterol levels, may predispose the pregnant woman to gallstone formation (cholelithiasis). Pain in the epigastric region following ingestion of a high-fat meal constitutes the major symptom of these conditions. The pain is self-limiting and usually resolves within 2 hours. b. The symptoms described are not associated with influenza. c. The symptoms described are not associated with urinary tract infection. d. Prolonged emptying time of bile from the gallbladder, combined with elevated blood cholesterol levels, make cholelithiasis a more probable diagnosis than indigestion. 22 10. The clinic nurse reviews the complete blood count results for a 30-year-old woman who is now 33 weeks’ gestation. Tamara’s hemoglobin value is 11.2 g/dL, and her hematocrit is 38%. The clinic nurse interprets these findings as: a. Normal adult values b. Normal pregnancy values for the third trimester c. Increased adult values d. Increased values for 33 weeks’ gestation ANS: b 11. The clinic nurse is aware that the pregnant woman’s blood volume increases by: a. 20% to 25% b. 30% to 35% c. 40% to 45% d. 50% to 55% ANS: c Feedback a. The values are low normal for adults but represent normal findings for pregnant women. b. During pregnancy the woman’s hematocrit values may appear low due to the increase in total plasma volume (on average, 50%). Because the plasma volume is greater than the increase in erythrocytes (30%), the hematocrit decreases by about 7%. This alteration is termed “physiologic anemia of pregnancy,” or “pseudo-anemia.” The hemodilution effect is most apparent at 32 to 34 weeks. The mean acceptable hemoglobin level in pregnancy is 11 to 12 g/dL of blood. c. The values are not increased; they are low normal for adults but represent normal findings for pregnant women. d. The values are not increased; they are low normal for adults but represent normal findings for pregnant women. Feedback a. An increase in maternal blood volume begins during the first trimester and peaks at term. The increase approaches 40% to 45%, not 20% to 25%. b. An increase in maternal blood volume begins during the first trimester and peaks at term. The increase approaches 40% to 45, not 30% to 35%. 25 ANS: a Feedback a. Nutrition and weight management play an essential role in the development of a healthy pregnancy. Not only does the patient need to have an understanding of the essential nutritional elements, she must also be able to assess and modify her diet for the developing fetus and her own nutritional maintenance. To facilitate this process, it is the nurse’s responsibility to provide education and counseling concerning dietary intake, weight management, and potentially harmful nutritional practices. To facilitate this process, it is the nurse’s responsibility to gather more information on the woman’s dietary practices through a food diary. b. Nutrition and weight management play an essential role in the development of a healthy pregnancy. To facilitate this process, it is the nurse’s responsibility to provide education and counseling concerning dietary intake, weight management, and potentially harmful nutritional practices. c. Nutrition and weight management play an essential role in the development of a healthy pregnancy. Not only does the patient need to have an understanding of the essential nutritional elements, she must also be able to assess and modify her diet for the developing fetus and her own nutritional maintenance. To facilitate this process, it is the nurse’s responsibility to provide education and counseling concerning dietary intake, weight management, and potentially harmful nutritional practices, not just inform the patient of expected normal weight gain. d. Nutrition and weight management play an essential role in the development of a healthy pregnancy. Not only does the patient need to have an understanding of the essential nutritional elements, she must also be able to assess and modify her diet for the developing fetus and her own nutritional maintenance. To 26 facilitate this process, it is the nurse’s responsibility to provide education and counseling concerning dietary intake, weight management, and potentially harmful nutritional practices. 15. A woman presents to a prenatal clinic appointment at 10 weeks’ gestation, in the first trimester of pregnancy. Which of the following symptoms would be considered a normal finding at this point in pregnancy? a. Occipital headache b. Urinary frequency c. Diarrhea d. Leg cramps ANS: b 16. The nurse is providing prenatal teaching to a group of diverse pregnant women. One woman, who indicates she smokes two to three cigarettes a day, asks about its impact on her pregnancy. The nurse explains that the most significant risk to the fetus is: a. Respiratory distress at birth b. Severe neonatal anemia c. Low neonatal birth weight d. Neonatal hyperbilirubinemia ANS: C Feedback a. Headaches may be benign or, especially if noted after 20 weeks’ gestation, may be a symptom of pregnancy-induced hypertension (PIH). b. Urinary frequency is a common complaint of women during their first trimester. c. Diarrhea is rarely seen in pregnancy. Constipation is a common complaint. d. Leg cramps are commonly seen during the second and third trimesters. Feedback a. Respiratory distress is not the most significant risk to the fetus unless the fetus is also premature. b. Severe neonatal anemia is not associated with pregnancies complicated by cigarette smoking. c. Low neonatal birth weight is the most common complication seen in pregnancies complicated by cigarette smoking. d. Neonatal hyperbilirubinemia is not associated 27 with pregnancies complicated by cigarette smoking. 17. While performing Leopold’s maneuvers on a woman in early labor, the nurse palpates a flat area in the fundal region, a hard-round mass on the left side, a soft round mass on the right side, and small parts just above the symphysis. The nurse concludes which of the following? a. The fetal position is right occiput posterior. b. The fetal attitude is flexed. c. The fetal presentation is scapular. d. The fetal lie is vertical. ANS: c Feedback a. This is a shoulder presentation. b. It is not possible to determine whether the attitude is flexed or not when doing Leopold’s maneuvers. c. This is a shoulder presentation. d. The lie is transverse or horizontal. 18. A nurse is reviewing diet with a pregnant woman in her second trimester. Which of the following foods should the nurse advise the patient to avoid consuming during her pregnancy? a. Brie cheese b. Bartlett pears c. Sweet potatoes d. Grilled lamb ANS: a 19. The nurse is working in a prenatal clinic caring for a patient at 14 weeks’ gestation, G2 P1001. Which of the following findings should the nurse highlight for the nurse midwife? a. Body mass index of 23 b. Blood pressure of 100/60 c. Hematocrit of 29% d. Pulse rate of 76 bpm Feedback a. Soft cheese may harbor Listeria. The patient should avoid consuming uncooked soft cheese. b. A pear is an excellent food for a pregnant woman to consume. c. Sweet potatoes are an excellent food for a pregnant woman to consume. d. Grilled lamb is an excellent food for a pregnant woman to consume, although it should be well cooked. 30 b. Epistaxis c. Bradycardia d. Oliguria ANS: b 25. A primigravida patient is 39 weeks pregnant. Which of the following symptoms would the nurse expect the patient to exhibit? a. Nausea b. Dysuria c. Urinary frequency d. Intermittent diarrhea ANS: c 26. The nurse has taken a health history on four multigravida patients at their first prenatal visits. It is high priority that the patient whose first child was diagnosed with which of the following diseases receives nutrition counseling? a. Development dysplasia of the hip b. Achondroplastic dwarfism c. Spina bifida d. Muscular dystrophy Feedback a. Diplopia is sometimes seen in patients with pregnancy-induced hypertension (PIH). b. Epistaxis is commonly seen in pregnant patients. The bleeding is related to the increased vascularity of the mucous membranes. Unless the blood loss is significant, it is a normal finding. c. Bradycardia is often seen immediately after delivery but not during the third trimester. d. Oliguria is seen in patients with PIH. Feedback a. Nausea is usually not seen in the third trimester. b. Dysuria is not a normal finding at any time during a pregnancy. The possibility of a urinary traction infection (UTI) should be considered. c. Urinary frequency recurs at the end of the third trimester. As the uterus enlarges, it again compresses the bladder causing urinary frequency. d. Diarrhea is not a normal finding at any time during a pregnancy. 31 ANS: c Feedback a. The etiology of developmental dysplasia of the hip is unrelated to the mother’s nutritional status. b. Achondroplasia is an inherited defect. Its etiology is unrelated to the mother’s nutritional status. c. The incidence of spina bifida is much higher in women with poor folic acid intakes. It is a priority that this patient receives nutrition counseling. d. Most forms of muscular dystrophy are inherited. Their etiologies are unrelated to the mother’s nutritional status. 27. A nurse working in a prenatal clinic is caring for a woman who asks advice on foods that are high in vitamin C because “I hate oranges.” The nurse states that 1 cup of which of the following raw foods will meet the patient’s daily vitamin C needs? a. Strawberries b. Asparagus c. Iceberg lettuce d. Cucumber ANS: a 28. The nurse notes each of the following findings in a woman at 10 weeks’ gestation. Which of the findings would enable the nurse to tell the woman that she is probably pregnant? a. Fetal heart rate via Doppler b. Positive pregnancy test c. Positive ultrasound assessment d. Absence of menstrual period ANS: b Feedback a. Strawberries are an excellent source of vitamin C. b. Although asparagus has some vitamin C, it is not an excellent source. c. Iceberg lettuce is a poor source of vitamin C. d. Cucumber is a poor source of vitamin C. Feedback a. A fetal heart rate is a positive sign of pregnancy. b. A positive pregnancy test is a probable sign of pregnancy. It is not a positive sign because the 32 hormone tested for—human chorionic gonadatropin (hCG)—may be being produced by, for example, a hydatidiform mole. c. A positive ultrasound is a positive sign of pregnancy. d. Amenorrhea is a presumptive sign of pregnancy. 29. A nurse who is discussing serving sizes of foods with a new prenatal patient would state that which of the following is equal to 1 (one) serving from the dairy food group? a.1 cup low-fat milk b. ½ cup vanilla yogurt c. ½ cup cottage cheese d.1-ounce cream cheese ANS: a Feedback a. 1 cup of any milk (e.g., whole milk, skim milk, buttermilk, chocolate milk) is equal to 1 serving size from the dairy group. b. 1 cup of yogurt is equal to 1 serving size from the dairy group. c. 1 ½ cup of cottage cheese is equal to 1 serving size from the dairy group. d. Cream cheese is not included in the dairy group. It is a fat product. 30. The nurse who is assessing a G2 P1 palpates the fundal height at the location noted on the picture below. 35 ANS: a, b, d Sweating is a warning sign that often precedes syncope. Syncope (a trandient loss of consciousness and postural tone with spontaneous recovery) during pregnancy is frequently attributed to orthostatic hypotension or inferior vena cava compression by the gravid uterus. Nausea and yawning are warning signs that often precede syncope. Lightheadedness, sweating, nausea, yawning, and feelings of warmth are warning signs that often precede syncope. Chills are not a warning sign that often precede syncope. 34. The perinatal nurse teaches the student nurse about the physiological changes in pregnancy that most often contribute to the increased incidence of urinary tract infections. These changes include (select all that apply): a. Relaxation of the smooth muscle of the urinary sphincter b. Relaxation of the smooth muscle of the bladder c. Inadequate emptying of the bladder d. Increased incidence of bacteriuria ANS: a, b, c, d Ascension of bacteria into the bladder can cause asymptomatic bacteriuria (ASB), or urinary tract infections (UTIs). These infections occur more frequently in pregnancy due to relaxation of the smooth muscle of the bladder and urinary sphincter and inadequate emptying of the bladder, changes that allow bacterial ascent into the bladder. 35. The clinic nurse discusses normal bladder function in pregnancy with a 22-year-old pregnant woman who is now in her 29th gestational week. The nurse explains that at this time in pregnancy, it is normal to experience (select all that apply): a. Urinary frequency b. Urinary urgency c. Nocturia d. Incontinence ANS: a, b, c During pregnancy, the bladder, a pelvic organ, is compressed by the weight of the growing uterus. The added pressure, along with progesterone-induced relaxation of the urethra and sphincter musculature, leads to urinary urgency, frequency, and nocturia. Incontinence of urine is not a normal change during pregnancy. 36. A 32-year-old woman now at 32 weeks’ gestation is complaining of right-sided sharp abdominal pain. The patient is examined by the clinic nurse and given information about abdominal discomfort in pregnancy. She is also instructed to seek immediate attention if she (select all that apply): a. Has heartburn b. Has chills or a fever c. Feels decreased fetal movements d. Has increased abdominal pain 36 ANS: b, c, d Heartburn is a common discomfort throughout pregnancy. Because the appendix is pushed upward and posterior by the gravid uterus, the typical location of pain is not a reliable indicator for a ruptured appendix during pregnancy. The pain should gradually subside, but if it persists or is accompanied by fever, a change in bowel habits, or decreased fetal movement, the patient should promptly contact her medical provider. 37. The clinic nurse talks with Suzy, a pregnant woman at 9 weeks’ gestation who has just learned of her pregnancy. Suzy’s nausea and vomiting are most likely caused by (select all that apply): a. Increased levels of estrogen b. Increased levels of progesterone c. An altered carbohydrate metabolism d. Increased levels of human chorionic gonadotropin ANS: c, d Nausea and vomiting during the first trimester most likely are related to rising levels of human chorionic gonadotropin (hCG) and altered carbohydrate metabolism. Changes in taste and smell, due to alterations in the oral and nasal mucosa, can further aggravate the gastrointestinal discomfort. 38. The clinic nurse encourages all pregnant women to increase their water intake to at least 8 to 10 glasses per day in order to (select all that apply): a. Decrease the risk of constipation b. Decrease the risk of bile stasis c. Decrease their feelings of fatigue d. Decrease the risk of urinary tract infections ANS: a, b, c, d Patients should be encouraged to drink at least 8 to 10 glasses of water each day and empty their bladders at least every 2 to 3 hours and immediately after intercourse. These measures will help prevent stasis of urine and the bacterial contamination that leads to infection, as well as constipation. Some women experience symptoms of fatigue that can be alleviated by remaining adequately hydrated. 39. The perinatal nurse examines the thyroid gland as part of the physical examination of Savannah, a pregnant woman who is now at 16 weeks’ gestation. The perinatal nurse informs Savannah that during pregnancy (select all that apply): a. Increased size of the thyroid gland is normal b. Increased function of the thyroid gland is normal c. Decreased function of the thyroid gland is normal d. The thyroid gland will return to its normal size and function during the postpartal period ANS: a, b, d 37 The thyroid gland changes in size and activity during pregnancy. Enlargement is caused by increased circulation from the progesterone-induced effects on the vessel walls, and by estrogen- induced hyperplasia of the glandular tissue. The thyroid gland increases not decreases in size and activity during pregnancy. The thyroid gland returns to normal size and activity postpartum. 40. The clinic nurse describes the respiratory system changes common to pregnancy to the new nurse. These changes include (select all that apply): a. An increased tidal volume b. A decreased airway resistance c. An increased chest circumference d. An increased airway resistance ANS: a, b, c During pregnancy, a number of changes occur to meet the woman’s increased oxygen requirements. The tidal volume (amount of air breathed in each minute) increases 30% to 40%. The enlarging uterus creates an upward pressure that elevates the diaphragm and increases the subcostal angle. The chest circumference may increase by as much as 6 centimeters, and airway resistance decreases. Although the “up and down” capacity of diaphragmatic movement is reduced, lateral movement of the chest and intercostal muscles accommodates for this loss of movement and keeps pulmonary functions stable. There is no increase in airway resistance during pregnancy. 41. The clinic nurse teaches the new nurse about pregnancy-induced blood clotting changes. The nurse explains that a pregnant woman is at risk for venous thrombosis due to (select all that apply): a. Increased fibrinogen volume b. Increased blood factor V c. Increased blood factor X d. Venous stasis ANS: a, c, d Although the platelet cell count does not change significantly during pregnancy, fibrinogen volume has been shown to increase by as much as 50%. This alteration leads to an increase in the sedimentation rate. Blood factors VII, VIII, IX, and X are also increased, and this change causes hypercoagulability. The hypercoagulability state, coupled with venous stasis (poor blood return from the lower extremities) places the pregnant woman at an increased risk for venous thrombosis, embolism, and, when complications are present, disseminated intravascular coagulation (DIC). Blood factor V does not increase. 42. The clinic nurse describes possible interventions for the pregnant woman who is experiencing pain and numbness in her wrists. The nurse suggests (select all that apply): a. Elevating the arms and wrists at night b. Reassessment during the postpartum period c. The use of “cock splints” to prevent wrist flexion d. Massaging the hands and wrists with alcohol 40 a. Dysuria b. Hematuria c. Urgency d. Delayed urination ANS: a, b, c Urinary tract infection (UTI) symptoms include dysuria, hematuria, and urgency. 50. Urinary tract infection (UTI) prevention measures during pregnancy include counseling the pregnant woman to (select all that apply): a. Delay urination until bladder is full b. Limit hydration c. Wipe from front to back d. Urinate after intercourse ANS: a, c, d Anticipatory guidance for urinary tract infection prevention includes delaying urination, wipe front to back, and maintaining adequate hydration. 51. Interventions for low back pain during pregnancy should include (select all that apply): a. Utilizing proper body mechanics b. Applying ice or heat to affected area c. Avoiding pelvic rock and pelvic tilt d. Using additional pillows for support during sleep ANS: a, b, d Interventions for back pain during pregnancy include utilizing proper body mechanics, applying heat or ice to the area, using additional pillows during sleep, and not avoiding pelvic rock/tilt, but encouraging pelvic rock/tilt. 52. Jorgina is a 24-year-old pregnant woman at 26 weeks’ gestation. This is Jorgina’s third pregnancy, and her obstetrical history includes one full-term birth, one preterm birth, and two living children. Today Jorgina arrives at the clinic with complaints of fatigue, insomnia, and backache. She reports that she is a nurse on an oncology unit and is worried about continuing with working her 12-hour shifts. The perinatal nurse identifies concerns in Jorgina’s history and work environment including (select all that apply): a. Risk of preterm birth b. Presence of chemotherapeutic agents c. Requirement for heavy lifting d. History of diabetes ANS: a, b, c Women who are currently experiencing pregnancy complications and those who have a history of pregnancy complications (such as history of preterm birth) or other preexisting health disorders may be required to reduce their hours or stop working. The potential for maternal exposure to toxic substances such as chemotherapeutic agents, lead, and ionizing radiation 41 (found in laboratories and health-care facilities); heavy lifting; and use of heavy machinery and other hazardous equipment should prompt reassignment to a different work area. If reassignment is not possible, Jorgina may need to stop working until the pregnancy has been completed. In this scenario there is no history of diabetes. 53. The clinic nurse is assessing the complete blood count results for Kim-Ly, a 23-year-old pregnant woman. Kim-Ly’s hemoglobin is 9.8 g/dL. This laboratory finding places Kim-Ly’s pregnancy at risk for (select all that apply): a. Preterm birth b. Placental abruption c. Intrauterine growth restriction d. Thrombocytopenia ANS: a, c True anemia, or iron-deficiency anemia, occurs when the hemoglobin level drops below 10 g/dL. The blood’s decreased oxygen-carrying capacity causes a reduction in oxygen transport to the developing fetus. Decreased fetal oxygen transport has been associated with intrauterine growth restriction (IUGR) and preterm birth. There is not a risk factor for abruption or thrombocytopenia. 54. Teera is a 22-year-old woman who is experiencing her third pregnancy. Her obstetrical history includes one first-trimester elective abortion and one first-trimester spontaneous abortion. Teera is a semi-vegetarian who drinks milk and eats yogurt and fish as part of her daily intake. The perinatal nurse discusses Teera’s diet with her as she may be deficient in (select all that apply): a. Iron b. Magnesium c. Zinc d. Vitamin B12 ANS: a, c Semi-vegetarian diets include fish, poultry, eggs, and dairy products but no beef or pork and have adequate intake of magnesium. Pregnant women who adhere to this diet may consume inadequate amounts of iron and zinc. Because strict vegetarians (vegans) consume only plant products, their diets are deficient in vitamin B12, found only in foods of animal origin. 55. During the initial antenatal visit, the clinic nurse asks questions about the woman’s nutritional intake. Specific questions should include information pertaining to (select all that apply): a. Preferred foods b. The presence of cravings c. Use of herbal supplements d. Aversions to certain foods and odors ANS: a, b, c, d 42 The nurse should obtain a nutritional history on all pregnant patients and patients of childbearing age to gain specific information related to the pregnancy, including foods that are preferred while pregnant (which may provide information about cultural and environmental dietary factors), special diets (which will assist the nurse in planning for education or interventions for risk factors associated with dietary practices), cravings or aversions to specific foods, and use of herbal supplements. 56. The perinatal nurse talks to the prenatal class attendees about guidelines for exercise in pregnancy. Recommended guidelines include (select all that apply): a. Stopping if the woman is tired b. Bouncing and slowly arching the back c. Increasing fluid intake throughout the physical activity d. Maintaining the ability to walk and talk during exercise ANS: a, c, d Women should adhere to some basic safety guidelines when formulating their exercise program, including monitoring the breathing rate and ensuring that the ability to walk and talk comfortably is maintained during physical activity, stopping exercise when the woman becomes tired, and maintaining adequate fluid intake. Pregnant women should avoid exercises that can cause any degree of trauma to the abdomen or those that include rigorous bouncing, arching of the back, or bending beyond a 45-degree angle. Short Answer 57. Lesions at the gum line that bleed easily ANS: Epulis gravidarum 58. Anterior convexity of the lumbar spine ANS: Lumbar lordosis Refer To: Glossary 59. Increased saliva production ANS: Ptyalism Refer To: Glossary 60. Reflux of the stomach contents into the esophagus ANS: Pyrosis Refer To: Glossary 61. Severe itching due to stasis of bile in the liver ANS: Pruritis gravidarum Refer To: Glossary 45 72. The clinic nurse monitors the blood pressure and assesses a woman’s urine at each prenatal visit to assess for signs or symptoms of . A previous history or the presence of a are also risk factors. ANS: preeclampsia; new partner A previous history of preeclampsia increases the woman’s likelihood of a recurrence during subsequent pregnancies. If a woman did not experience preeclampsia with previous pregnancies but has a new partner for her current pregnancy, her risk of developing preeclampsia is similar to that of a woman who is pregnant for the first time. Although preeclampsia is a systemic disorder that occurs only during pregnancy, it is generally recognized by two classic symptoms: elevated blood pressure and proteinuria. 73. The clinic nurse is aware of the importance of chlamydia screening during pregnancy. Chlamydia transmission to the infant at may result in . ANS: birth; ophthalmia neonatorum Chlamydia trachomatis is a bacterium that causes infection that is prevalent in sexually active populations, especially those in the under-25 age group. Complications of chlamydia infections include salpingitis, pelvic inflammatory disease, infertility, ectopic pregnancy, premature rupture of the membranes, and preterm birth. Transmission to the neonate may occur during birth and results in ophthalmia neonatorum and chlamydial neonatal pneumonia. 74. The prenatal nurse describes the need for and screening at the first antenatal visit. If the pregnant woman is not immune, she will be counseled to avoid contact with young children who have a rash and could be infectious. ANS: rubella; varicella Some of the routine maternal laboratory tests screen for childhood diseases that are known to cause congenital anomalies or other pregnancy complications if contracted during early pregnancy. When contracted during the first trimester, rubella causes a number of fetal deformities. Varicella (chickenpox) is another common childhood disease that may cause problems in the developing embryo and fetus. Therefore, all pregnant women are screened for rubella and varicella. 75. The prenatal nurse cautions a pregnant woman about Caesar salad consumption during pregnancy or any source of or milk. ANS: raw eggs; unpasteurized A word of caution should be provided by health-care providers to pregnant women with regard to microbial food-borne illness. Raw, or unpasteurized, milk as well as partially cooked eggs and foods containing raw or partially cooked eggs should be avoided. 76. The clinic nurse describes to the student nurse that is excessive saliva production in pregnancy. This condition is most likely caused by increased levels. 46 ANS: ptyalism; hormone Ptyalism, or excessive salivation, can be quite distressing for the pregnant woman who must frequently wipe her mouth or spit into a cup. Although the cause of ptyalism is unknown, it is most likely related to increased hormone levels. 77. The clinic nurse talks with the newly diagnosed pregnant woman about the nausea that the woman is experiencing in this pregnancy. The clinic nurse suggests eating meals more often, remaining after eating, and the using techniques. ANS: smaller; upright; relaxation Nausea is often one of the first symptoms of pregnancy experienced. Nurses can suggest strategies to help offset the nausea, such as the avoidance of “trigger foods” (foods that cause nausea from sight or smell) and tight clothing that constricts the abdomen. The use of relaxation techniques (i.e., slow, deep breathing, mental imagery) can also help to decrease nausea. Other techniques that are often helpful include consuming plain, dry crackers or sucking on peppermint candy before arising; adhering to small, frequent meals; and remaining in an upright position after eating. 78. The clinic nurse understands that the physiological changes of pregnancy include vascular relaxation from the effects of and impaired venous circulation from pressure exerted by the enlarged uterus, predisposing the pregnant woman to . ANS: progesterone; varicose veins Progesterone results in vascular relaxation which combined with impaired venous return increases the incidence of varicose veins in pregnant women. 79. The perinatal nurse knows that , which is the eating of nonnutritive substances, is a common . ANS: pica; eating disorder Pica, the consumption of nonnutritive substances or food, is a common eating disorder that can affect pregnancy. Substances that are most often ingested include clay, dirt, cornstarch, and ice. Matching The clinic nurse understands the meaning of the following terms related to pregnancy care. Match these terms with the definitions listed below: Advocacy Lordosis Amenorrhea Ballottement Striae gravidarum Preterm birth 80. Passive movement of the unengaged fetus 47 ANS: Ballottement Refer To: Glossary 81. Verbalizing someone else’s wishes if he or she is unable to do so ANS: Advocacy Refer To: Chapter 2 82. Absence of menses ANS: Amenorrhea Refer To: Glossary 83. Curvature of the spine ANS: Lordosis Refer To: Glossary 84. Stretch marks ANS: Striae gravidarum Refer To: Glossary Chapter 5: Psycho-Social-Cultural Aspects of the Antepartum Period Multiple Choice 1. Sally is in her third trimester and has begun to sing and talk to the fetus. Sally is probably exhibiting signs of: a. Mental illness b. Delusions c. Attachment d. Crisis ANS: c Feedback a. This is normal maternal–fetal adaptation. b. Delusions are not real, and the fetus is real. c. Correct, because talking to the fetus is a sign of positive maternal adaptation. All other answers indicate pathology. d. Interacting with the fetus in utero represents normal development of attachment to the fetus. 2. What is the most common expected emotional reaction of a woman to the news that she is pregnant? a. Jealousy b. Acceptance c. Ambivalence d. Depression 50 d. This is not appropriate for a 2-year-old but may be appropriate for older age groups. 7. The nurse is interviewing a pregnant client who states she plans to drink chamomile tea to ensure an effective labor. The nurse knows that this is an example of: a. Cultural prescription b. Cultural taboo c. Cultural restriction d. Cultural demonstration ANS: a Feedback a. Correct. Cultural prescription is an expected behavior of the pregnant woman during the childbearing period. b. Taboos are cultural restrictions believed to have serious supernatural consequences. Drinking chamomile tea would not be in this category. c. Restrictions are activities during the childbearing period which are limited for the pregnant woman. Drinking chamomile tea would not be in this category. d. Demonstration is not a term that is used in relation to cultural behaviors. 8. Which of the following would be a priority for the nurse when caring for a pregnant woman who has recently emigrated from another country? a. Help her develop a realistic, detailed birth plan. b. Identify her support system. c. Teach her about expected emotional changes of pregnancy. d. Refer her to a doula for labor support. ANS: b Feedback a. A detailed birth plan may not be culturally appropriate and is not first priority. b. Correct, because lack of social support has been correlated with an increased risk of pregnancy complications and difficult adaptation to pregnancy. Pregnant women who are recent immigrants face many challenges in obtaining needed social support, and the nurse should first identify her support system to plan further interventions and referrals. c. There may be cultural variations in emotional changes of pregnancy. d. The nurse should first identify her support system before planning further interventions and referrals. 51 9. A pregnant client at 20 weeks’ gestation comes to the clinic for her prenatal visit. Which of the following client statements would indicate a need for further assessment? a. “I hate it when the baby moves.” b. “I’ve started calling my mom every day.” c. “My partner and I can’t stop talking about the baby.” d. “I still don’t know much time I’m going to take off work after the baby comes.” ANS: a Feedback a. Experiencing quickening as unpleasant may be a sign of maladaptation to pregnancy and needs further assessment by the nurse. b. This is an expected finding in maternal adaptation and development of the maternal role. c. This is an expected finding in maternal adaptation and development of the maternal role. d. At 20 weeks’ gestation, the client still has plenty of time to process this decision. 10. A pregnant client asks the nurse why she should attend childbirth classes. The nurse’s response would be based on which of the following information? a. Attending childbirth class is a good way to make new friends. b. Childbirth classes will help new families develop skills to meet the challenges of childbirth and parenting. c. Attending childbirth classes will help a pregnant woman have a shorter labor. d. Childbirth classes will help a pregnant woman decrease her chance of having a cesarean delivery. ANS: b Feedback a. There may be a beneficial effect of childbirth classes, but this is not the primary goal of childbirth education. b. Correct. These are the stated goals of childbirth education (ICEA, Lamaze). c. Evidence remains inconclusive regarding linking attendance at childbirth classes with a decreased incidence of cesarean section and shorter labors. d. Evidence remains inconclusive regarding linking attendance at childbirth classes with a decreased incidence of cesarean section and shorter labors. 11. A woman presents for prenatal care at 6 weeks’ gestation by LMP. Which of the following 52 findings would the nurse expect to see? a. Multiple pillow orthopnea b. Maternal ambivalence c. Fundus at the umbilicus d. Pedal and ankle edema ANS: b 12. A first-time father is experiencing couvade syndrome. He is likely to exhibit which of the following symptoms or behaviors? a. Urinary frequency b. Hypotension c. Bradycardia d. Prostatic hypertrophy ANS: a 13. When providing a psychosocial assessment on a pregnant woman at 21 weeks’ gestation, the nurse would expect to observe which of the following signs? a. Ambivalence b. Depression c. Anxiety d. Happiness ANS: d Feedback a. Orthopnea is a common complaint of women during the third trimester. b. Ambivalence is a common feeling of women during the first trimester. c. The fundus should be at the umbilicus at 20 weeks’ gestation. d. Dependent edema is a common complaint of women during the third trimester. Feedback a. Urinary frequency is a common symptom of couvade. b. The father’s blood pressure is not usually affected. c. The father’s heart rate is not usually affected. d. Prostatic changes are not related to couvade. Feedback a. Ambivalence is often seen during the first trimester. 55 often surface with the recognition that he is not as excited about the pregnancy as his partner, and couvade syndrome, the experience of maternal signs and symptoms, may develop. c. This cluster of symptoms is indicative of couvade syndrome, the experience of maternal signs and symptoms of pregnancy. d. This cluster of symptoms is indicative of couvade syndrome, the experience of maternal signs and symptoms of pregnancy. Multiple Response 18. The clinic nurse encourages paternal attachment during pregnancy by including the father in (select all that apply): a. Prenatal visits b. Ultrasound appointments c. Prenatal class information d. History taking and obtaining prenatal screening information ANS: b, c, d Pregnancy is psychologically stressful for men; some enjoy the role of nurturer, but others feel alienated and begin to stray from the relationship. The nurse can be instrumental in promoting early paternal attachment. Involvement of the father during examinations and tests and prenatal classes, along with thorough explanations of the need for them, can minimize the father’s feelings of being left out. A history and prenatal screening should be conducted at the first prenatal visit with the woman alone to ensure confidentiality and an open discussion of any problems or concerns she may have. The history should include information about the current pregnancy; the obstetric and gynecologic history; and a cultural assessment, and a medical, nutritional, social, and family (including the father’s) medical history. 19. The perinatal nurse screens all pregnant women early in pregnancy for maternal attachment risk factors, which include (select all that apply): a. Adolescence b. Low educational level c. History of depression d. A strong support system for the pregnancy ANS: a, b, c Maternal attachment to the fetus is an important area to assess and can be useful in identifying families at risk for maladaptive behaviors. The nurse should assess for indicators such as unintended pregnancy, domestic violence, difficulties in the partner relationship, sexually transmitted infections, limited financial resources, substance use, adolescence, poor social 56 support systems, low educational level, the presence of mental conditions, or adolescence that might interfere with the patient’s ability to bond with and care for the infant. A strong support system can facilitate the patient’s ability to bond with and care for the infant. 20. Strategies for culturally responsive care include (select all that apply): a. Practicing ethnocentrism b. Applying stereotyping c. Examining one’s own biases d. Learning another language ANS: c, d The only actions among the choices that are culturally responsive are examining one’s own biases and learning another language. Ethnocentrism and stereotyping are not culturally responsive actions. Fill-in-the-Blank 21. The clinic nurse talks with Becky, a 16-year-old woman who is now 28 weeks’ gestation. Today’s visit is only the second prenatal appointment that Becky has kept. The nurse wonders if Becky’s failure to come for routine prenatal checks is, in part, related to an adolescent’s orientation to the , rather than to the . ANS: present; future The adolescent may not seek prenatal care unless pressured by authority figures or peers to do so. By nature, adolescents are not future oriented. Hence, the pregnant adolescent may not be able to readily accept the reality of the unborn child. 22. According to Rubin, the mother-to-be needs to accept the pregnancy and incorporate it into her own reality and . This process is known as “ .” ANS: self-concept; binding in The mother-to-be needs to accept the pregnancy and incorporate it into her own reality and self- concept. This process is known as “binding in.” Acceptance of the child is critical to a successful adjustment to the pregnancy. Acceptance must come from the expectant woman as well as from others. 23. The clinic nurse asks pregnant women about their acceptance and planning for this pregnancy as a component of domestic violence screening. The nurse is aware that a(n) pregnancy the risk for domestic violence. ANS: unplanned; increases Intimate partner violence (IPV) may occur for the first time during pregnancy, or the nurse may identify evidence during the physical examination that is suspicious of ongoing physical abuse. Acceptance of pregnancy may be delayed if it was unplanned or unwanted. As a women’s advocate, nurses have a duty to be observant, to actively listen, and to use communication skills to gain clarification and understanding. 57 Chapter 6: Antepartal Tests Multiple Choice 1. Your pregnant patient is in her first trimester and is scheduled for an abdominal ultrasound. When explaining the rationale for early pregnancy ultrasound, the best response is: a. “The test will help to determine the baby’s position.” b. “The test will help to determine how many weeks you are pregnant.” c. “The test will help to determine if your baby is growing appropriately.” d. “The test will help to determine if you have a boy or girl.” ANS: b Feedback a. Fetal position during pregnancy changes, and position in the first trimester is not indicative of position later in pregnancy. b. Fetal growth and size are fairly consistent during the first trimester and are a reliable indicator of the weeks of gestation. c. Fetal growth is best assessed later in pregnancy. d. The primary rationale for ultrasounds is not to determine gender. 2. Your pregnant patient is having maternal alpha-fetoprotein (AFP) screening. She does not understand how a test on her blood can indicate a birth defect in the fetus. The best reply by the nurse is: a. “We have done this test for a long time.” b. “If babies have a neural tube defect, alpha-fetoprotein leaks out of the fetus and is absorbed into your blood, causing your level to rise. This serum blood test detects that rise.” c. “Neural tube defects are a genetic anomaly, and we examine the amount of alpha-fetoprotein in your DNA.” d. “If babies have a neural tube defect, this results in a decrease in your level of alpha-fetoprotein.” ANS: b Feedback a. This response does not explain AFP screening. b. When a neural tube defect is present, AFP is absorbed in the maternal circulation, resulting in a rise in the maternal AFP level. c. AFP testing is not related to DNA. d. Fetal neural tube defects result in an increase in maternal AFP. 3. The primary complications of amniocentesis are: a. Damage to fetal organs b. Puncture of umbilical cord c. Maternal pain d. Infection 60 a. Pain at the puncture site b. Macular rash on the abdomen c. Decrease in urinary output d. Cramping of the uterus ANS: d 8. A laboratory report indicates the L/S ratio (lecithin/sphingomyelin) results from an amniocentesis of a gravid patient with preeclampsia are 2:1. The nurse interprets the result as which of the following? a. The baby’s lung fields are mature. b. The mother is high risk for hemorrhage. c. The baby’s kidneys are functioning poorly. d. The mother is high risk for eclampsia. ANS: a Feedback a. An L/S ratio of 2:1 usually indicates that the fetal lungs are mature. b. L/S ratios are unrelated to maternal blood loss. c. L/S ratios are unrelated to fetal renal function. d. L/S ratios are unrelated to maternal risk for becoming eclamptic. Chapter 7: High-Risk Antepartum Nursing Care Multiple Choice 1. A client on 2 gm/hr. of magnesium sulfate has decreased deep tendon reflexes. Identify the priority nursing assessment to ensure client safety. a. Assess uterine contractions continuously. b. Assess fetal heart rate continuously. c. Assess urinary output. d. Assess respiratory rate. ANS: d Feedback a. Monitoring contractions does not indicate Feedback a. It is normal for the patient to experience pain at the puncture site. b. A rash is not an expected complication. c. Oliguria is not an expected complication. d. The woman should report any uterine cramping. Although rare, amniocentesis could stimulate preterm labor. 61 magnesium toxicity. b. Magnesium sulfate will decrease fetal variability and not provide an accurate assessment of magnesium toxicity. c. Urinary output does not correlate to decreased deep tendon reflexes. d. Correct. Respiratory effort and deep tendon reflexes (DTRs) are involuntary, and a decrease in DTRs could indicate the risk of magnesium sulfate toxicity and the risk for decreased respiratory effort. 2. A pregnant client with a history of multiple sexual partners is at highest risk for which of the following complications: a. Premature rupture of membranes b. Gestational diabetes c. Ectopic pregnancy d. Pregnancy-induced hypertension ANS: c Feedback a. Multiple partners do not increase a woman’s risk of premature rupture of membranes. b. Genetics and client diet and weight are contributing factors to gestational diabetes. c. Correct. A history of multiple sexual partners places the client at a higher risk of having contracted a sexually transmitted disease that could have ascended the uterus to the fallopian tubes and caused fallopian tube blockage, placing the client at high risk for an ectopic pregnancy. d. Multiple sexual partners are not a risk factor for pregnancy-induced hypertension. 3. Identify the hallmark of placenta previa that differentiates it from abruptio placenta. a. Sudden onset of painless vaginal bleeding b. Board-like abdomen with severe pain c. Sudden onset of bright red vaginal bleeding d. Severe vaginal pain with bright red bleeding ANS: a Feedback a. Correct. When the placenta attaches to the lower uterine segment near or over the cervical os, bleeding may occur without the onset of contractions or pain. b. The hallmark for abruptio placenta is pain and a board-like abdomen. c. Bright red bleeding could be related to abruptio placenta, placenta previa, or other 62 complications of pregnancy. d. Pain is not a hallmark of placenta previa. 4. Which of the following assessments would indicate instability in the client hospitalized for placenta previa? a. BP <90/60 mm/Hg, Pulse <60 BPM or >120 BPM b. FHR moderate variability without accelerations c. Dark brown vaginal discharge when voiding d. Oral temperature of 99.9 F ANS: a Feedback a. A decrease in BP accompanied by bradycardia or tachycardia is an indication of hypovolemic shock. b. FHR with moderate variability can be absent of accelerations during fetal sleep cycles or after maternal sedation. c. Bright red vaginal bleeding is an indication of current bleeding. d. Oral temperature may fluctuate based on the client’s hydration status. It should be reassessed. Cause for concern is a temperature of 100.4 F or more. 5. During pregnancy, poorly controlled asthma can place the fetus at risk for: a. Hyperglycemia b. IUGR c. Hypoglycemia d. Macrosomia ANS: b 6. Which of the following nursing diagnoses is of highest priority for a client with an ectopic pregnancy who has developed disseminated intravascular coagulation (DIC)? a. Risk for deficient fluid volume b. Risk for family process interrupted c. Risk for disturbed identity d. High risk for injury ANS: a Feedback a. Maternal asthma does not place the fetus at risk for hyperglycemia. b. Compromised pulmonary function can lead to decompensation and hypoxia that decrease oxygen flow to the fetus and can cause intrauterine growth restriction (IUGR). c. Asthma does not directly affect glycemic control. d. A fetus experiencing hypoxia would be small for gestational age, not large for gestational age. 65 11. The perinatal nurse is assessing a woman in triage who is 34 + 3 weeks’ gestation in her first pregnancy. She is worried about having her baby “too soon,” and she is experiencing uterine contractions every 10 to 15 minutes. The fetal heart rate is 136 beats per minute. A vaginal examination performed by the health-care provider reveals that the cervix is closed, long, and posterior. The most likely diagnosis would be: a. Preterm labor b. Term labor c. Back labor d. Braxton-Hicks contractions ANS: d 12. The perinatal nurse knows that the term to describe a woman at 26 weeks’ gestation with a history of elevated blood pressure who presents with a urine showing 2+ protein (by dipstick) is: a. Preeclampsia b. Chronic hypertension c. Gestational hypertension d. Chronic hypertension with superimposed preeclampsia ANS: d Feedback a. Preterm labor (PTL) is defined as regular uterine contractions and cervical dilation before the end of the 36th week of gestation. Many patients present with preterm contractions, but only those who demonstrate changes in the cervix are diagnosed with preterm labor. b. Term labor occurs after 37 weeks’ gestation. c. There is no indication in this scenario that this is back labor. d. Braxton-Hicks contractions are regular contractions occurring after the third month of pregnancy. They may be mistaken for regular labor, but unlike true labor, the contractions do not grow consistently longer, stronger, and closer together, and the cervix is not dilated. Some patients present with preterm contractions, but only those who demonstrate changes in the cervix are diagnosed with preterm labor. Feedback a. Preeclampsia is a multisystem, vasopressive disease process that targets the cardiovascular, hematologic, hepatic, and renal and central nervous systems. 66 b. Chronic hypertension is hypertension that is present and observable prior to pregnancy or hypertension that is diagnosed before the 20th week of gestation. c. Gestational hypertension is a nonspecific term used to describe the woman who has a blood pressure elevation detected for the first time during pregnancy, without proteinuria. d. The following criteria are necessary to establish a diagnosis of superimposed preeclampsia: hypertension and no proteinuria early in pregnancy (prior to 20 weeks’ gestation) and new-onset proteinuria, a sudden increase in protein—urinary excretion of 0.3 g protein or more in a 24-hour specimen, or two dipstick test results of 2+ (100 mg/dL), with the values recorded at least 4 hours apart, with no evidence of urinary tract infection; a sudden increase in blood pressure in a woman whose blood pressure has been well controlled; thrombocytopenia (platelet count lower than 100,000/mmC); and an increase in the liver enzymes alanine transaminase (ALT) or aspartate transaminase (AST) to abnormal levels. 13. A patient is receiving magnesium sulfate for severe preeclampsia. The nurse must notify the attending physician immediately of which of the following findings? a. Patellar and biceps reflexes of +4 b. Urinary output of 50 mL/hr c. Respiratory rate of 10 rpm d. Serum magnesium level of 5 mg/dL ANS: c Feedback a. The magnesium sulfate has been ordered because the patient has severe pregnancy- induced hypertension. Patellar and biceps reflexes of +4 are symptoms of the disease. b. The urinary output must be above 25 mL/hr. c. The drop in respiratory rate may indicate that the patient is suffering from magnesium toxicity. The nurse should report the finding to the physician. d. The therapeutic range of magnesium is 4 to 7 mg/dL. 67 14. A woman in labor and delivery is being given subcutaneous terbutaline for preterm labor. Which of the following common medication effects would the nurse expect to see in the mother? a. Serum potassium level increases b. Diarrhea c. Urticaria d. Complaints of nervousness ANS: d 15. Which of the following signs or symptoms would the nurse expect to see in a woman with concealed abruptio placentae? a. Increasing abdominal girth measurements b. Profuse vaginal bleeding c. Bradycardia with an aortic thrill d. Hypothermia with chills ANS: a 16. A woman who has had no prenatal care was assessed and found to have hydramnios on admission to the labor unit and has since delivered a baby weighing 4500 grams. Which of the following complications of pregnancy likely contributed to these findings? a. Pyelonephritis b. Pregnancy-induced hypertension c. Gestational diabetes d. Abruptio placentae Feedback a. The nurse would not expect to see a rise in the mother’s serum potassium levels. b. The beta agonists are not associated with diarrhea. c. The beta agonists are not associated with urticaria. d. Complaints of nervousness are commonly made by women receiving subcutaneous beta agonists. Feedback a. The nurse would expect to see increasing abdominal girth measurements. b. Profuse vaginal bleeding is rarely seen in placental abruption and is never seen when the abruption is concealed. c. With excessive blood loss, the nurse would expect to see tachycardia. d. The nurse would expect to see a stable temperature. 70 21. A woman at 32 weeks’ gestation is diagnosed with severe preeclampsia with HELLP syndrome. The nurse will identify which of the following as a positive patient care outcome? a. Rise in serum creatinine b. Drop in serum protein c. Resolution of thrombocytopenia d. Resolution of polycythemia ANS: c 22. A 16-year-old patient is admitted to the hospital with a diagnosis of severe preeclampsia. The nurse must closely monitor the woman for which of the following? a. High leukocyte count b. Explosive diarrhea c. Fractured pelvis d. Low platelet count ANS: d Feedback a. A rise in serum creatinine indicates that the kidneys are not effectively excreting creatinine. It is a negative outcome. b. A drop in serum protein indicates that the kidneys are allowing protein to be excreted. This is a negative outcome. c. Resolution of thrombocytopenia is a positive sign. It indicates that the platelet count is returning to normal. d. Polycythemia is not related to HELLP syndrome. Rather one sees a drop in red cell and platelet counts with HELLP. A positive sign, therefore, would be a rise in the RBC count. Feedback a. High leukocyte count is not associated with severe pregnancy-induced hypertension (PIH) or HELLP (hemolysis, elevated liver enzymes, and low platelets) syndrome. b. Explosive diarrhea is not associated with severe PIH or HELLP syndrome. c. A fractured pelvis is not associated with severe PIH or HELLP syndrome. d. Low platelet count is one of the signs associated with HELLP (hemolysis, elevated liver enzymes, and low platelets) syndrome. 71 23. A woman at 10 weeks’ gestation is diagnosed with gestational trophoblastic disease (hydatiform mole). Which of the following findings would the nurse expect to see? a. Platelet count of 550,000/ mm3 b. Dark brown vaginal bleeding c. White blood cell count 17,000/ mm3 d. Macular papular rash ANS: b 24. After an education class, the nurse overhears an adolescent woman discussing safe sex practices. Which of the following comments by the young woman indicates that additional teaching about sexually transmitted infection (STI) control issues is needed? a. “I could get an STI even if I just have oral sex.” b. “Girls over 16 are less likely to get STDs than younger girls.” c. “The best way to prevent an STI is to use a diaphragm.” d. “Girls get human immunodeficiency virus (HIV) easier than boys do.” ANS: c Feedback a. The nurse would not expect to see an elevated platelet count. b. The nurse would expect to see dark brown vaginal discharge c. The nurse would not expect to see an elevated white blood cell count. d. The nurse would not expect to see a rash. Feedback a. This statement is true. Organisms that cause sexually transmitted infections can invade the respiratory and gastrointestinal tracts. b. This statement is true. Young women are especially high risk for becoming infected with sexually transmitted diseases. c. This statement is untrue. The young woman needs further teaching. Condoms protect against STDs and pregnancy. In addition, condoms can be kept in readiness for whenever sex may occur spontaneously. Using condoms does not require the teen to plan to have sex. A diaphragm is not an effective infection-control method. Plus, it would require the teen to plan for intercourse. d. This statement is true. Young women are higher risk for becoming infected with HIV than are young men. 72 25. A woman who is admitted to labor and delivery at 30 weeks’ gestation, is 1 cm dilated, and is contracting q 5 minutes. She is receiving magnesium sulfate IV piggyback. Which of the following maternal vital signs is most important for the nurse to assess each hour? a. Temperature b. Pulse c. Respiratory rate d. Blood pressure ANS: c 26. You are caring for a patient who was admitted to labor and delivery at 32 weeks’ gestation and diagnosed with preterm labor. She is currently on magnesium sulfate, 2 gm per hour. Upon your initial assessment you note that she has a respiratory rate of 8 with absent deep tendon reflexes. What will be your first nursing intervention? a. Elevate head of the bed b. Notify the MD c. Discontinue magnesium sulfate d. Draw a serum magnesium level ANS: c Initial nursing intervention needs to be discontinuing magnesium sulfate because the patient is exhibiting signs of magnesium toxicity with absent deep tendon reflexes and decreased respiratory rate. 27. A 34-weeks’ gestation multigravida, G3 P1 is admitted to the labor suite. She is contracting every 7 minutes and 40 seconds. The woman has several medical problems. Which of the following of her comorbidities is most consistent with the clinical picture? a. Kyphosis b. Urinary tract infection c. Congestive heart failure d. Cerebral palsy ANS: b Feedback a. The temperature should be monitored, but it is not the most important vital sign. b. The pulse rate should be monitored, but it is not the most important vital sign. c. The respiratory rate is the most important vital sign. Respiratory depression is a sign of magnesium toxicity. d. The blood pressure should be monitored, but it is not the most important vital sign. Feedback a. Kyphosis is unrelated to preterm labor. b. Urinary tract infections often precipitate 75 for a magnesium sulfate infusion. 32. The perinatal nurse is providing care to Marilyn, a 25-year-old G1 TPAL 0000 woman hospitalized with severe hypertension at 33 weeks’ gestation. The nurse is preparing to administer the second dose of beta-methasone prescribed by the physician. Marilyn asks: “What is this injection for again?” The nurse’s best response is: a. “This is to help your baby’s lungs to mature.” b. “This is to prepare your body to begin the labor process.” c. “This is to help stabilize your blood pressure.” d. “This is to help your baby grow and develop in preparation for birth.” ANS: a 33. A woman who is 36 weeks pregnant presents to the labor and delivery unit with a history of congestive heart disease. Which of the following findings should the nurse report to the primary health-care practitioner? a. Presence of chloasma b. Presence of severe heartburn c. 10-pound weight gain in a month d. Patellar reflexes +1 ANS: c Feedback a. Antenatal glucocorticoids such as beta- methasone may be given (12 mg IM 24 hours apart) to promote fetal lung maturity if the gestational age is less than 34 weeks and childbirth can be delayed for 48 hours. b. Antenatal glucocorticoids such as beta- methasone may be given (12 mg IM 24 hours apart) to promote fetal lung maturity if the gestational age is less than 34 weeks and childbirth can be delayed for 48 hours. c. Antenatal glucocorticoids such as beta- methasone may be given (12 mg IM 24 hours apart) to promote fetal lung maturity if the gestational age is less than 34 weeks and childbirth can be delayed for 48 hours. d. Antenatal glucocorticoids such as beta- methasone may be given (12 mg IM 24 hours apart) to promote fetal lung maturity if the gestational age is less than 34 weeks and childbirth can be delayed for 48 hours. Feedback a. Chloasma is a normal pregnancy finding. b. Heartburn is an expected finding during the 76 third trimester. c. The weight gain may be due to fluid retention. Fluid retention may occur in patients with pregnancy-induced hypertension and in patients with congestive heart failure. The physician should be notified. d. Although slightly hyporeflexic, patellar reflexes of +1 are within normal limits. 34. The single most important risk factor for preterm birth includes: a. Uterine and cervical anomalies b. Infection c. Increased BMI d. Prior preterm birth ANS: d The single most important factor is prior preterm birth with a reoccurrence rate of up to 40%. 35. Your antepartal patient is 38 weeks’ gestation, has a history of thrombosis, and has been on strict bed rest for the last 12 hours. She is now experiencing shortness of breath. What about the patient may be a contributing factor for her shortness of breath? a. Physiologic changes in pregnancy result in vasodilation, which increases the tendency to form blood clots. b. Physiologic changes in pregnancy result in vasoconstriction, which increases the tendency to form blood clots. c. Physiologic changes in pregnancy result in anemia, which increases the tendency to form blood clots. d. Physiologic changes in pregnancy result in decreased perfusion to the lungs, which increases the tendency to form blood clots. ANS: a The patient’s shortness of breath, bed rest, and history of thrombosis indicate possible pulmonary embolism. Her pregnant state also increases the potential for thrombosis resulting from increased levels of coagulation factors and decreased fibrinolysis, venous dilation, and obstruction of the venous system by the gravid uterus. Thromboembolitic diseases occurring most frequently in pregnancy include deep vein thrombosis and pulmonary embolism. 36. Metabolic changes during pregnancy glucose tolerance. a. lower b. increase c. maintain d. alter ANS: a Metabolic changes during pregnancy lower glucose tolerance. 77 True/False 37. Immediately postpartum, the insulin needs in diabetic women increase dramatically. ANS: FalseThere is a significant decrease in the need for insulin immediately after delivery related to the loss of antagonistic placental hormones and suppression of the anterior pituitary growth hormone. 38. The perinatal nurse observes the placental inspection by the health-care provider after birth. This examination may help to determine whether an abruption has occurred prior to or during labor. ANS: True Fifty percent of abruptions occur before labor and after the 30th week, 15% occur during labor, and 30% are identified only upon inspection of the placenta after delivery. 39. It is critical for the perinatal nurse to learn, as part of the facility’s policies and procedures, to immediately perform a vaginal examination on a woman who presents with vaginal bleeding after 24 weeks’ gestation. ANS: False Placenta previa should be suspected in all patients who present with bleeding after 24 completed weeks of gestation. Because of the risk of placental perforation, vaginal examinations are not performed. 40. The perinatal nurse knows that the survival rate for infants born at or greater than 28 to 29 gestational weeks is greater than 90%. ANS: True With appropriate medical care, neonatal survival dramatically improves as the gestational age increases, with over 50% of neonates surviving at 25 weeks’ gestation, and over 90% surviving at 28 to 29 weeks of gestation. 41. A patient with hypertension who is receiving intravenous magnesium sulfate therapy has requested an epidural anesthetic. The perinatal nurse should first review the patient’s complete blood count results for evidence of a decreased platelet count. ANS: True Baseline information, including complete blood count (CBC), clotting studies, serum electrolytes, and renal function tests, is used to alert the care providers to changes in the patient’s condition as additional laboratory tests are obtained. 42. The perinatal nurse knows that the laboring diabetic patient’s blood glucose level should always be less than 120 mg/dL. ANS: True 80 transport to a facility equipped with a neonatal intensive care unit. 46. The perinatal nurse provides a hospital tour for couples and families preparing for labor and birth in the future. Teaching is an important component of the tour. Information provided about preterm labor and birth prevention includes (select all that apply): a. Encouraging regular, ongoing prenatal care b. Reporting symptoms of urinary frequency and burning to the health-care provider c. Coming to the labor triage unit if back pain or cramping persist or become regular d. Lying on the right side, withholding fluids, and counting fetal movements if contractions occur every 5 minutes ANS: a, b, c Feedback a. The nurse should encourage all pregnant women to obtain prenatal care and screen for vaginal and urogenital infections and treat appropriately, and remind pregnant women to call their provider repeatedly if symptoms of preterm labor occur. b. Educating all women of childbearing age about preterm labor is a crucial component of prevention. The nurse should encourage all pregnant women to obtain prenatal care and screen for vaginal and urogenital infections and treat appropriately, and remind pregnant women to call their provider repeatedly if symptoms of preterm labor occur. c. Educating all women of childbearing age about preterm labor is a crucial component of prevention. The nurse should encourage all pregnant women to obtain prenatal care and screen for vaginal and urogenital infections and treat appropriately and remind pregnant women to call their provider if symptoms of preterm labor occur. d. Lying on the right side; drinking fluids, not withholding fluids; and counting fetal movements if contractions occur every 5 minutes are recommended if a woman thinks she is contracting. 47. The perinatal nurse describes for the new nurse the various risks associated with prolonged premature preterm rupture of membranes. These risks include (select all that apply): a. Chorioamnionitis b. Abruptio placentae 81 c. Operative birth d. Cord prolapse ANS: a, b, d Even though maintaining the pregnancy to gain further fetal maturity can be beneficial, prolonged PPROM has been correlated with an increased risk of chorioamnionitis, placental abruption, and cord prolapse. 48. Betamethasone is a steroid that is given to a pregnant woman with signs of preterm labor. The purpose of giving steroids is to (select all that apply): a. Stimulate the production of surfactant in the preterm infant b. Be given between 24 and 34 weeks’ gestation c. Increase the severity of respiratory distress d. Accelerate fetal lung maturity ANS: a, b, d Betamethasone is a steroid that is given to pregnant women with signs of preterm labor between 24 and 34 weeks’ gestation. It stimulates the production of surfactant in the preterm infant and accelerates fetal lung maturity. 49. Marked hemodynamic changes in pregnancy can impact the pregnant woman with cardiac disease. Signs and symptoms of deteriorating cardiac status include (select all that apply): a. Orthopnea b. Nocturnal dyspnea c. Palpitations d. Irritation ANS: a, b, c Signs and symptoms of deteriorating cardiac status with cardiac disease include orthopnea, nocturnal dyspnea, and palpitations, but do not include irritation. Short Answer 50. A condition where the placenta attaches to the lower uterine segment of the uterus ANS: Placenta previa 51. A pregnancy that ends before 20 weeks’ gestation ANS: Miscarriage 52. Birth prior to 37 completed weeks of pregnancy ANS: Preterm birth 82 53. Specks or spots in the vision where the patient cannot see; “blind spots” ANS: Scotoma 54. A disease characterized by an abnormal placental development that results in the production of fluid-filled grapelike clusters and a vast proliferation of trophoblastic tissue ANS: Hydatidiform mole/Gestational trophoblastic disease 55. No expulsion of the products of conception, but bleeding and dilation of the cervix such that a pregnancy is unlikely ANS: Inevitable abortion 56. Placement of suture to mechanically close a weak cervix ANS: Cervical cerclage Fill-in-the-Blank 57. The perinatal nurse knows that an early pregnancy loss occurs before weeks, and a late pregnancy loss is one that occurs between 12 and weeks. ANS: 12; 20 Not all conceptions result in a live-born infant. Of all clinically recognized pregnancies, 10% to 20% are lost, and approximately 22% of pregnancies detected on the basis of hCG assays are lost before the appearance of any clinical signs or symptoms. By definition, an early pregnancy loss occurs before 12 weeks of gestation; a late pregnancy loss is one that occurs between 12 and 20 weeks. 58. Mary, a G3 TPAL 0020 woman at 20 weeks’ gestation, has had a transvaginal ultrasound. Mary has been informed that she has cervical incompetence. The perinatal nurse explains that this diagnosis means that her cervix has without contractions. ANS: dilated; regular Patients with cervical incompetence usually present with painless dilation and effacement of the cervix, often during the second trimester of pregnancy. The patient frequently gives a history of repeated second trimester losses with no apparent etiology. Incompetent cervix is estimated to cause approximately 15% of all second trimester losses. 59. The perinatal nurse knows that nausea and vomiting are common in pregnancy and usually resolve by weeks’ gestation. The severe form of this condition is . 85 discharge. d. True labor contractions bring about changes in cervical effacement and dilation, and with false labor there are irregular contractions with little or no cervical changes. ANS: d Feedback a. Rupture of membranes can occur prior to labor or during labor. b. A woman’s response to labor may not be reflective of her status in labor but is influenced by expectations and emotional status. c. Loss of the mucus plug can occur prior to the onset of labor. d. True labor contractions bring about changes in cervical effacement and dilation, and with false labor there are irregular contractions with little or no cervical changes. 4. The mechanism of labor known as cardinal movements of labor are the positional changes that the fetus goes through to best navigate the birth process. These cardinal movements are: a. Engagement, Descent, Flexion, Extension, Internal rotation, External rotation, Expulsion b. Engagement, Descent, Flexion, Internal rotation, Extension, External rotation, Expulsion c. Engagement, Flexion, Internal rotation, Extension, External rotation, Descent, Expulsion d. Engagement, Flexion, Internal rotation, Extension, External rotation, Flexion, Expulsion ANS: b Feedback a. The order of the cardinal movements is incorrect. b. Engagement occurs when the greatest diameter of the fetal head passes through the pelvic inlet. Engagement can occur late in pregnancy or early in labor. Descent is the movement of the fetus through the birth canal during the first and second stages of labor. Flexion is when the chin of the fetus moves toward the fetal chest. Flexion occurs when the descending head meets resistance from maternal tissues. This movement results in the smallest fetal diameter to the maternal pelvic dimensions. It typically occurs early in labor. Internal rotation is the movement, the rotation of the fetal head, that aligns the long axis of the fetal head with the long axis of the maternal pelvis. It occurs mainly during the second stage of labor. Extension is the movement facilitated by 86 resistance of the pelvic floor, causing the presenting part to pivot beneath the pubic symphysis and the head to be delivered. This occurs during the second stage of labor. External rotation is when the sagittal suture moves to a transverse diameter and the shoulders align in the anteroposterior diameter. The sagittal suture maintains alignment with the fetal trunk as the trunk navigates through the pelvis. Expulsion is the movement that occurs when the shoulders and remainder of the body are delivered. c. The order of the cardinal movements is incorrect. d. The order of the cardinal movements is incorrect. 5. A woman is considered in active labor when: a. Cervical dilation progresses from 4 to 7 cm with effacement of 40% to 80%, contractions become more intense, occurring every 2 to 5 minutes with duration of 45 to 60 seconds. b. Cervical dilation progresses to 3 cm with effacement of 30, contractions become more intense, occurring every 2 to 5 minutes with duration of 45 to 60 seconds. c. Cervical dilation progresses to 8 cm with effacement of 80%, contractions become more intense, occurring every 2 to 5 minutes with duration of 45 to 60 seconds. d. Cervical dilation progresses to 10 cm with effacement of 90%, contractions become more intense, occurring every 2 to 5 minutes with duration of 45 to 60 seconds. ANS: a Feedback a. Characteristics of this phase are the cervix dilates, on an average, 1.2 cm/hr for primiparous women and 1.5 cm/hr for multiparous women. Cervical dilation progresses from 4 to 7 cm with effacement of 40% to 80%. Fetal descent continues, and contractions become more intense, occurring every 2 to 5 minutes with duration of 45 to 60 seconds, and discomfort increases. b. Cervical dilation progresses to 3 cm with effacement of 30, indicating the early or latent phase of labor. c. Cervical dilation progresses to 8 cm with effacement of 80%, indicating the transition phase of labor. d. Cervical dilation of 10 cm with effacement is the end of the first stage of labor. 87 6. You are caring for a woman in labor who is 6 cm dilated with a reassuring FHT pattern and regular strong UCs. The fetal heart rate (FHR) should be: a. Monitored continuously b. Monitored every 15 minutes c. Monitored every 30 minutes d. Monitored every 60 minutes ANS: c Feedback a. Assessment of fetal heart rate (FHR) during the active phase of labor with a reassuring FHR is not indicated continuously. b. Assessment of fetal heart rate (FHR) during the active phase of labor with a reassuring FHR is not indicated every 15 minutes. c. Assessment of fetal heart rate (FHR) during the active phase of labor with a reassuring FHR is indicated every 30 minutes. d. Assessment of fetal heart rate (FHR) during the active phase of labor with a reassuring FHR is indicated every 30 minutes, not every 60 minutes. 7. A woman you are caring for in labor requests an epidural for pain relief in labor. Included in your preparation for epidural placement is a baseline set of vital signs. The most common vital sign to change after epidural placement: a. Blood pressure, hypotension b. Blood pressure, hypertension c. Pulse, tachycardia d. Pulse, bradycardia ANS: a Feedback a. Blood pressure, hypotension, as up to 40% of women may experience hypotension. Hypotension is defined as systolic BP <100 mm Hg or 20% decrease in BP from preanesthesia levels. Intravenous bolus is typically given to decrease the incidence of hypotension. b. Blood pressure, hypertension is incorrect because hypotension is the common complication after epidural placement. c. Pulse, tachycardia is incorrect because hypotension is the common complication after epidural placement. d. Pulse, bradycardia is incorrect because hypotension is the common complication after epidural placement. 90 c. There is no indication in this scenario that this is back labor. d. Braxton-Hicks contractions are regular contractions occurring after the third month of pregnancy. They may be mistaken for regular labor, but unlike true labor, the contractions do not grow consistently longer, stronger, and closer together, and the cervix is not dilated. Some patients present with preterm contractions, but only those who demonstrate changes in the cervix are diagnosed with preterm labor. 12. The perinatal nurse knows that the term to describe a woman at 26 weeks’ gestation with a history of elevated blood pressure who presents with a urine showing 2+ protein (by dipstick) is: a. Preeclampsia b. Chronic hypertension c. Gestational hypertension d. Chronic hypertension with superimposed preeclampsia ANS: d Feedback a. Preeclampsia is a multisystem, vasopressive disease process that targets the cardiovascular, hematologic, hepatic, and renal and central nervous systems. b. Chronic hypertension is hypertension that is present and observable prior to pregnancy or hypertension that is diagnosed before the 20th week of gestation. c. Gestational hypertension is a nonspecific term used to describe the woman who has a blood pressure elevation detected for the first time during pregnancy, without proteinuria. d. The following criteria are necessary to establish a diagnosis of superimposed preeclampsia: hypertension and no proteinuria early in pregnancy (prior to 20 weeks’ gestation) and new-onset proteinuria, a sudden increase in protein—urinary excretion of 0.3 g protein or more in a 24-hour specimen, or two dipstick test results of 2+ (100 mg/dL), with the values recorded at least 4 hours apart, with no evidence of urinary tract infection; a sudden increase in blood pressure in a woman whose blood pressure has been well controlled; 91 thrombocytopenia (platelet count lower than 100,000/mmC); and an increase in the liver enzymes alanine transaminase (ALT) or aspartate transaminase (AST) to abnormal levels. 13. A patient is receiving magnesium sulfate for severe preeclampsia. The nurse must notify the attending physician immediately of which of the following findings? a. Patellar and biceps reflexes of +4 b. Urinary output of 50 mL/hr c. Respiratory rate of 10 rpm d. Serum magnesium level of 5 mg/dL ANS: c 14. A woman in labor and delivery is being given subcutaneous terbutaline for preterm labor. Which of the following common medication effects would the nurse expect to see in the mother? a. Serum potassium level increases b. Diarrhea c. Urticaria d. Complaints of nervousness ANS: d Feedback a. Magnesium sulfate has been ordered because the patient has severe pregnancy-induced hypertension. Patellar and biceps reflexes of +4 are symptoms of the disease. b. The urinary output must be above 25 mL/hr. c. The drop in respiratory rate may indicate that the patient is suffering from magnesium toxicity. The nurse should report the finding to the physician. d. The therapeutic range of magnesium is 4 to 7 mg/dL. Feedback a. The nurse would not expect to see a rise in the mother’s serum potassium levels. b. The beta agonists are not associated with diarrhea. c. The beta agonists are not associated with urticaria. d. Complaints of nervousness are commonly made by women receiving subcutaneous beta agonists. 92 15. Which of the following signs or symptoms would the nurse expect to see in a woman with concealed abruptio placentae? a. Increasing abdominal girth measurements b. Profuse vaginal bleeding c. Bradycardia with an aortic thrill d. Hypothermia with chills ANS: a 16. A woman who has had no prenatal care was assessed and found to have hydramnios on admission to the labor unit and has since delivered a baby weighing 4500 grams. Which of the following complications of pregnancy likely contributed to these findings? a. Pyelonephritis b. Pregnancy-induced hypertension c. Gestational diabetes d. Abruptio placentae ANS: c 17. For the patient with which of the following medical problems should the nurse question a physician’s order for beta agonist tocolytics? a. Type 1 diabetes mellitus b. Cerebral palsy c. Myelomeningocele d. Positive group B streptococci culture Feedback a. The nurse would expect to see increasing abdominal girth measurements. b. Profuse vaginal bleeding is rarely seen in placental abruption and is never seen when the abruption is concealed. c. With excessive blood loss, the nurse would expect to see tachycardia. d. The nurse would expect to see a stable temperature. Feedback a. Pyelonephritis does not lead to the development of hydramnios or macrosomia. b. Pregnancy-induced hypertension does not lead to the development of hydramnios or macrosomia. c. Untreated gestational diabetics often have hydramnios and often deliver macrosomic babies. d. Abruptio placentae does not lead to the development of hydramnios or macrosomia. 95 and platelet counts with HELLP. A positive sign, therefore, would be a rise in the RBC count. 22. A 16-year-old patient is admitted to the hospital with a diagnosis of severe preeclampsia. The nurse must closely monitor the woman for which of the following? a. High leukocyte count b. Explosive diarrhea c. Fractured pelvis d. Low platelet count ANS: d 23. A woman at 10 weeks’ gestation is diagnosed with gestational trophoblastic disease (hydatidiform mole). Which of the following findings would the nurse expect to see? a. Platelet count of 550,000/mm3 b. Dark brown vaginal bleeding c. White blood cell count 17,000/mm3 d. Macular papular rash ANS: b 24. After an education class, the nurse overhears an adolescent woman discussing safe sex practices. Which of the following comments by the young woman indicates that additional teaching about sexually transmitted infection (STI) control issues is needed? a. “I could get an STI even if I just have oral sex.” Feedback a. High leukocyte count is not associated with severe pregnancy-induced hypertension (PIH) or HELLP (hemolysis, elevated liver enzymes, and low platelets) syndrome. b. Explosive diarrhea is not associated with severe PIH or HELLP syndrome. c. A fractured pelvis is not associated with severe PIH or HELLP syndrome. d. Low platelet count is one of the signs associated with HELLP (hemolysis, elevated liver enzymes, and low platelets) syndrome. Feedback a. The nurse would not expect to see an elevated platelet count. b. The nurse would expect to see dark brown vaginal discharge. c. The nurse would not expect to see an elevated white blood cell count. d. The nurse would not expect to see a rash. 96 b. “Girls over 16 are less likely to get STDs than younger girls.” c. “The best way to prevent an STI is to use a diaphragm.” d. “Girls get human immunodeficiency virus (HIV) easier than boys do.” ANS: c 25. A woman who is admitted to labor and delivery at 30 weeks’ gestation, is 1 cm dilated, and is contracting q 5 minutes. She is receiving magnesium sulfate IV piggyback. Which of the following maternal vital signs is most important for the nurse to assess each hour? a. Temperature b. Pulse c. Respiratory rate d. Blood pressure ANS: c Feedback a. This statement is true. Organisms that cause sexually transmitted infections can invade the respiratory and gastrointestinal tracts. b. This statement is true. Young women are especially high risk for becoming infected with sexually transmitted diseases. c. This statement is untrue. The young woman needs further teaching. Condoms protect against STDs and pregnancy. In addition, condoms can be kept in readiness for whenever sex may occur spontaneously. Using condoms does not require the teen to plan to have sex. A diaphragm is not an effective infection-control method. Plus, it would require the teen to plan for intercourse. d. This statement is true. Young women are higher risk for becoming infected with HIV than are young men. Feedback a. The temperature should be monitored, but it is not the most important vital sign. b. The pulse rate should be monitored, but it is not the most important vital sign. c. The respiratory rate is the most important vital sign. Respiratory depression is a sign of magnesium toxicity. d. The blood pressure should be monitored, but it is not the most important vital sign. 97 26. A primiparous woman has been admitted at 35 weeks’ gestation and diagnosed with HELLP syndrome. Which of the following laboratory changes is consistent with this diagnosis? a. Hematocrit dropped to 28%. b. Platelets increased to 300,000 cells/mm3. c. Red blood cells increased to 5.1 million cells/mm3. d. Sodium dropped to 132 mEq/dL. ANS: a 27. A labor nurse is caring for a patient, 39 weeks’ gestation, who has been diagnosed with placenta previa. Which of the following physician orders should the nurse question? a. Type and cross-match her blood. b. Insert an internal fetal monitor electrode. c. Administer an oral stool softener. d. Assess her complete blood count. ANS: b Feedback a. It would be appropriate to type and cross- match the patient for a blood transfusion. b. This action is inappropriate. When a patient has a placenta previa, nothing should be inserted into the vagina. c. To prevent constipation, it is appropriate for a patient to take a stool softener. d. It is appropriate to monitor the patient for signs of anemia. 28. A type 1 diabetic patient has repeatedly experienced elevated serum glucose levels throughout her pregnancy. Which of the following complications of pregnancy would the nurse expect to see? a. Postpartum hemorrhage b. Neonatal hyperglycemia c. Postpartum oliguria d. Neonatal macrosomia ANS: d Feedback a. The nurse would expect to see a drop in the hematocrit: The H in HELLP stands for hemolysis. b. The nurse would expect to see low platelets. c. The nurse would expect to see hemolysis. d. The sodium is usually unaffected in HELLP syndrome. Feedback
Docsity logo



Copyright © 2024 Ladybird Srl - Via Leonardo da Vinci 16, 10126, Torino, Italy - VAT 10816460017 - All rights reserved